2017 AMC 10B Problems/Problem 25: Difference between revisions
Blanked the page |
No edit summary |
||
| Line 1: | Line 1: | ||
==Cheap Solution== | |||
By inspection, the sequences <math>91,93,92,96,98,100,95</math> and <math>93,91,92,96,98,100,95</math> work, so the answer is $\boxed{\textbf{E } 100}. | |||
Revision as of 08:57, 16 February 2017
Cheap Solution
By inspection, the sequences
and
work, so the answer is $\boxed{\textbf{E } 100}.